0 Daumen
923 Aufrufe

Aufgabe:

Hi, brauche bitte Hilfe bei folgender Aufgabe:

$$\begin{array} { c } { \text { Es sei } V = \mathbb { C } ^ { n } \text { und } A \in \mathbb { C } ^ { n \times n } \text { eine hermitesche Matrix. Zeigen Sie, dass die Abbildung } } \\ { \langle , \rangle _ { A } : V \times V \rightarrow \mathbb { C } , \langle x , y \rangle _ { A } : = \overline { x } ^ { T } \cdot A \cdot y } \\ { \text { eine hermitesche Sesquilinearform ist. } } \end{array}$$


Problem/Ansatz:

Wir haben in der Vorlesung bereits gezeigt: Ist $$\langle x , y \rangle _ { A }$$ eine Sesquilinearform, so gilt: $$\begin{array} { c }  { \langle x , y \rangle _ { A } = \overline { x } ^ { T } \cdot A \cdot y } \end{array}$$

Wie kann ich nun zeigen, dass $$\langle x , y \rangle _ { A }$$ auch hermitesch ist?

Danke schonmal ;)

Avatar von

1 Antwort

0 Daumen
 
Beste Antwort

" hermitesch "   heißt doch nur    <x,y> =  <y,x> konjugiert.

Mit dem Ergebnis aus der Vorlesung hast du doch

$$\begin{array} { c }  { \langle x , y \rangle _ { A } = \overline { x } ^ { T } \cdot A \cdot y } \end{array}$$

und

$$\begin{array} { c }  { \langle y , x \rangle _ { A } = \overline { y } ^ { T } \cdot A \cdot x } \end{array}$$


und wenn du die rechte Seite der 2. Gleichung  konjugierst bekommt du

die rechte Seite der ersten, weil A hermitesch ist.

Avatar von 288 k 🚀

Danke für die schnelle Antwort. Aber würde das nicht voraussetzen, dass die Aussage aus der Vorlesung kommutativ ist?

Danke für die schnelle Antwort.

Ich verstehe allerdings nicht, warum die 2. Gleichung konjugiert, die erste ergibt.

Könntest du das bitte etetw ausführlicher beschreiben?

Du musst doch nur zeigen

$$\begin{array} { c }  { \langle x , y \rangle _ { A } }=   \overline { { \langle y , x \rangle _ { A } }  }   \end{array}$$

Deshalb hatte ich das so geschrieben:

$$\begin{array} { c }  { \langle x , y \rangle _ { A } = \overline { x } ^ { T } \cdot A \cdot y } \end{array}$$und$$\begin{array} { c }  { \langle y , x \rangle _ { A } = \overline { y } ^ { T } \cdot A \cdot x } \end{array}$$

Jetzt fehlt für den Nachweis der Gleichung nur,

dass man die 2. Gleichung auf beiden Seiten konjugiert.

Sieht dann so aus:

$$\overline{ { \langle y , x \rangle _ { A }}}=\overline {\overline { y } ^ { T } \cdot A \cdot x } $$

$$={ y } ^ { T } \cdot \overline { A} \cdot \overline { x} $$

Und das ist ja jetzt eine Zahl, also sozusagen eine

1x1 Matrix, die kannst du einfach transponieren und hast dann:

$$=({ y } ^ { T } \cdot \overline { A} \cdot \overline { x})^T $$

und nach den Regeln für das Transponieren dreht sich dann die

Reihenfolge um und das Ergebnis passt, weil A hermitesch ist.

Jetzt hab ich es. Vielen Dank :))

Ein anderes Problem?

Stell deine Frage

Willkommen bei der Mathelounge! Stell deine Frage einfach und kostenlos

x
Made by a lovely community